Vous êtes sur la page 1sur 4

Volume 14, Number 4 December 2009-March, 2010

Olympiad Corner A Refinement of Bertrand’s Postulate


The 2010 Chinese Mathematical Neculai Stanciu
Olympiad was held on January. Here (Buzău, Romania)
are the problems.
Problem 1. As in the figure, two In this article, we give an elementary Theorem 2. For n ≥ 1, there is a prime
circles Γ1, Γ2 intersect at points A, B. A demonstration of the famous Bertrand’s number p such that n < p < 3(n+1)/2.
line through B intersects Γ1, Γ2 at C, D postulate by using a theorem proved by (Since 3(n+1)/2<2n for n > 3, this is a
respectively. Another line through B the mathematician M. El Bachraoni in refinement of the Bertrand’s postulate.)
intersects Γ1, Γ2 at E, F respectively. 2006.
Line CF intersects Γ1, Γ2 at P, Q For the proof, the case n=1 follows
Interesting is the distribution of from 1<p=2<3. The case n=2 follows
respectively. Let M, N be the midpoints
prime numbers among the natural from 2<p=3<9/2. For n even, say n=2k,
of arcs PB, arc QB respectively. Prove
numbers and problems about their by Theorem 1, we have a prime p such
that if CD = EF, then C, F, M, N are
distributions have been stated in very that n=2k <p < 3k <3(2k+1)/2=3(n+1)/2.
concyclic.
simple ways, but they all turned out to Similarly, for n odd, say n=2k+1, we
F be very difficult. The following open have a prime p such that n = 2k+1 <
problem was stated by the Polish 2k+2=2(k+1) < p < 3(k+1)=3(n+1)/2.
Γ mathematician W. Sierpiński in 1958:
A Γ 2
1 P
For all natural numbers n > 1 and k ≤ n, Concerning the distribution of prime
there is at least one prime in the range numbers among the natural numbers,
M
Q
E [kn,(k+1)n]. recently (in 2008) Rafael Jakimczuk has
N proved a formula (see [2] or [4]) for the
C
B D The case k=1 (known as Bertrand’s n-th prime pn, which provided a better
postulate) was stated in 1845 by the error term than previous known
Problem 2. Let k ≥ 3 be an integer. approximate formulas for pn. His
French mathematician J. Bertrand and
Sequence {an} satisfies ak=2k and for formula is for n≥4,
was proved by the Russian
all n > k, an= an−1 + 1 if an−1 and n are
mathematician P. L. Chebysev. Simple pn = n logn + log(n logn)(n − Li(n logn))
coprime and an=2n if an−1 and n are not
proofs have been given by the
coprime. Prove that the sequence ∞
(−1) k Qk −1 (log(n log n))
Hungarian mathematician P. Erdos in +∑ (n − Li (n log n)) k
{an−an−1} contains infinitely many k =2 k!n k −1 log k −1 n
1932 and recently by the Romanian
prime numbers. + O(h(n)), where
mathematician M. Tena [3]. The case
x
(continued on page 4) k=2 was proved in 2006 by M. El dt n log 2 n
Li ( x) = ∫ , h ( n) =
Editors: 張 百 康 (CHEUNG Pak-Hong), Munsang College, HK Bachraoni (see [1]). His proof was log t exp(d log n )
2
高 子 眉 (KO Tsz-Mei) relatively short and not too complicated.
梁 達 榮 (LEUNG Tat-Wing)
It is freely available on the internet [4].
李 健 賢 (LI Kin-Yin), Dept. of Math., HKUST and Qk−1(x) are polynomials.
吳 鏡 波 (NG Keng-Po Roger), ITC, HKPU
Below we will present a refinement
Artist: 楊 秀 英 (YEUNG Sau-Ying Camille), MFA, CU References
of Bertrand’s postulate and it is perhaps
Acknowledgment: Thanks to Elina Chiu, Math. Dept.,
HKUST for general assistance. the simplest demonstration of the [1] M. El Bachraoni, “Primes in the
On-line:
postulate based on the following Interval [2n,3n],” Int. J. Contemp.
http://www.math.ust.hk/mathematical_excalibur/ Math. Sciences, vol. 1, 2006, no. 13,
The editors welcome contributions from all teachers and Theorem 1. For any positive integer n > 617-621.
students. With your submission, please include your name, 1, there is a prime number between 2n
address, school, email, telephone and fax numbers (if
and 3n. (For the proof, see [1] or [4].) [2] R. Jakimczuk, “An Approximate
available). Electronic submissions, especially in MS Word,
are encouraged. The deadline for receiving material for the Formula for Prime Numbers,” Int. J.
next issue is April 17, 2010. The demonstration in [1] was typical Contemp. Math. Sciences, vol. 3, 2008,
For individual subscription for the next five issues for the of many theorems in number theory and no. 22, 1069-1086.
09-10 academic year, send us five stamped self-addressed
envelopes. Send all correspondence to: was based on multiple inequalities valid
[3] M. Tena, “O demonstraţie a
for large values of n which can be
Dr. Kin-Yin LI, Math Dept., Hong Kong Univ. of Science postulatului lui Bertrand,” G. M.-B 10,
and Technology, Clear Water Bay, Kowloon, Hong Kong calculated effectively. For the rest of the
2008.
Fax: (852) 2358 1643 values of n, there are many basic
Email: makyli@ust.hk improvisations, some perhaps difficult [4] http://www.m-hikari.com/ijcms.html
© Department of Mathematics, The Hong Kong University to follow.
of Science and Technology
Mathematical Excalibur, Vol. 14, No. 4, Dec. 09-Mar. 10 Page 2

Max-Min Inequalities For the first inequality, we observe that ⎛ 1⎞


2
⎛1 1 1⎞
⎜n + ⎟ ≥ (a1 + a2 +L+ an )⎜⎜ + +L+ ⎟⎟
1 1 1 1 bcd + acd + abd + abc ⎝ 2⎠ ⎝ a1 a2 an ⎠
Pedro Henrique O. Pantoja + + + =
a b c d abcd ⎛ 1 1 1⎞
(UFRN, NATAL, BRAZIL)
= bcd + acd + abd + abc.
= (m + a2 + L + M )⎜⎜ + + L + ⎟⎟
⎝ M a2 m ⎠
There are many inequalities. In this 2
Now, by the AM-GM inequality, we have ⎛ m M ⎞
article, we would like to introduce the ≥ ⎜⎜ +n−2+ ⎟.
readers to some inequalities that a3+b3+c3 ≥ 3abc, a3+b3+d3 ≥ 3abd, M m ⎟⎠

involve maximum and minimum. a3+c3+d3 ≥ 3acd and b3+c3+d3 ≥ 3bcd.
Adding these four inequalities, we get the Taking square root of both sides,
The first example was a problem first inequality.
from the Federation of Bosnia for
Grade 1 in 2008. 1 m M
n+ ≥ +n−2+ .
Next, let S=a+b+c+d. Then we have 2 M m
Example 1 (Bosnia-08) For arbitrary
real numbers x, y and z, prove the S = a + b + c + d ≥ 4(abcd )1 / 4 = 4 Simplifying, we get 2(m + M ) ≤ 5 mM .
following inequality: 3 2
by the AM-GM inequality and so S = S S Squaring both sides, we can get
x 2 + y 2 + z 2 − xy − yz − zx ≥ 16S. The second inequality follows by
⎧ 3( x − y ) 2 3( y − z ) 2 3( z − x) 2 ⎫ applying the power mean inequality to 4M 2−17mM+4m2 ≥ 0.
≥ max ⎨ , , ⎬. obtain
⎩ 4 4 4 ⎭
3
Factoring, we see
a3 + b3 + c3 + d 3 ⎛ a + b + c + d ⎞ S3 S
Solution. Without loss of generality, ≥⎜ ⎟ = ≥ .
4 ⎝ 4 ⎠ 64 4 (4M−m)(M−4m) ≥ 0.
suppose x ≥ y ≥ z. Then
Since 4M−m ≥ 0, we get M−4m ≥ 0,
⎧3(x − y)2 3( y − z)2 3(z − x)2 ⎫ 3
max⎨ , , ⎬ = (z − x) .
2
Example 3. Let a, b, c be positive real which is the desired inequality.
⎩ 4 4 4 ⎭ 4
numbers. Prove that if x = max{a,b,c}
and y = min{a,b,c}, then The next example was problem 1 on the
Let a = x−y, b = y−z and c = z−x.
Then c = −(a+b). Hence, (z−x)2 = c2 = 2008 Greek National Math Olympiad.
x y 18 abc
(a+b)2 = a2+2ab+b2 and + ≥ .
y x ( a + b + c )( a 2 + b 2 + c 2 ) Example 5. (Greece-08) For positive
x 2 + y 2 + z 2 − xy − yz − zx integers a1, a2, …, an, prove that if
Solution. Suppose a ≥ b ≥ c. Then x = a k=max{a1,a2,…,an} and t=min{a1,a2,…,
1
= [( x − y ) 2 + ( y − z ) 2 + ( z − x ) 2 ] and y = c. Using the AM-GM inequality an}, then
2
1 and the Cauchy-Schwarz inequality, we kn
= (a 2 + b 2 + a 2 + 2ab + b 2 ) have
2 ⎛ ∑ n ai2 ⎞ t n
⎜ i =1 ⎟ ≥
a c a 2 + c 2 (a 2 + c 2 )b
+ = = ⎜ n a ⎟ ∏ ai ,
= a + ab + b .
2 2
c a ac abc ∑
⎝ i =1 ⎠i i =1

So it suffices to show ( 2ac)b 54abc


≥ = When does equality hold?
[(a + b + c) / 3]3 ( a + b + c)3
3 Solution. By the Cauchy-Schwarz
a 2 + ab + b 2 ≥ (a 2 + 2ab + b 2 ), 54abc
4 ≥ . inequality,
3(a 2 + b 2 + c 2 )(a + b + c) 2
which is equivalent to (a−b)2 ≥ 0. ⎛ n ⎞ n n n
⎜ ∑ ai ⎟ ≤ ∑ 12 ∑ ai2 =n∑ ai2 .
⎝ i =1 ⎠ i =1 i =1 i =1
The next example was a problem on The next example was problem 4 in the
the 1998 Iranian Mathematical Hence,
Olympiad. 2009 USA Mathematical Olympiad.
∑ ∑
n n
ai2 ai
Example 2. (Iran-98) Let a, b, c, d be Example 4. (USAMO-09) For n ≥ 2, let a1,
i =1
≥ i =1
.

n
ai n
positive real numbers such that abcd=1. a2, …, an be positive real numbers such that i =1
Prove that Since each ai ≥ 1, the right side of the
2
⎛ ⎞ above inequality is at least one. Also, we
a +b +c +d (a1 + a2 +L+ an )⎜⎜ 1 + 1 +L+ 1 ⎟⎟ ≤ ⎛⎜n + 1 ⎞⎟ .
3 3 3 3

have kn/t ≥ n. So, applying the above


⎧ 1 1 1 1⎫
≥ max ⎨a + b + c + d , + + + ⎬. ⎝ a1 a2 an ⎠ ⎝ 2⎠
inequality and the AM-GM inequality
⎩ a b c d⎭ Prove that we have
Solution. It suffices to show max{a1,a2,…,an}≤ 4 min{a1, a2,…, an}. kn
n
⎛ ∑n ai2 ⎞ t ⎛ ∑n ai ⎞ n
⎜ i =1 ⎟ ≥ ⎜ i =1 ⎟ ≥
⎜ n ⎟ ∏
1 1 1 1 ai .
⎜ n a ⎟
a 3 + b3 + c3 + d 3 ≥ + + +
a b c d
Solution. Without loss of generality, we
may assume

⎝ i =1 ⎠
i ⎝ ⎠ i =1

and Equality holds if and only if all ai’s are


m=a1 ≤ a2 ≤ ⋯ ≤ an = M.
a + b + c + d ≥ a + b + c + d.
3 3 3 3 equal.
By the Cauchy-Schwarz inequality, (continued on page 4)
Mathematical Excalibur, Vol. 14, No. 4, Dec. 09-Mar. 10 Page 3

Problem Corner FURDUI (Campia Turzii, Cluj, Romania), ΔBFA. So AF/EF=BF/AF. Then
HUNG Ka Kin Kenneth (Diocesan
Boys’ School), LKL Problem Solving PF = AF = EF × BF .
We welcome readers to submit their Group (Madam Lau Kam Lung
solutions to the problems posed below Secondary School of MFBM), Paolo Comments: For those who are not
for publication consideration. The PERFETTI (Math Dept, Università degli aware of the incenter characterization
solutions should be preceded by the studi di Tor Vergata Roma, via della used above, they may see Math
solver’s name, home (or email) address ricerca scientifica, Roma, Italy). Excalibur, vol. 11, no. 2 for details.
and school affiliation. Please send Let ω = cos(π/n) + i sin(π/n). Then we Other commended solvers: CHOW
submissions to Dr. Kin Y. Li, have ωn = −1 and (ωk + ω−k)/2 = cos(kπ/n). Tseung Man (True Light Girls’
Department of Mathematics, The Hong So College), CHUNG Ping Ngai (La
Kong University of Science & n Salle College, Form 6), Nicholas
n −1 n−1
⎛ ω k + ω −k ⎞
Technology, Clear Water Bay, Kowloon, ∑(−1) k
cosn (kπ / n) = ∑ω kn ⎜⎜
2
⎟⎟ LEUNG (St. Paul’s School, London)
Hong Kong. The deadline for sending k =0 k =0 ⎝ ⎠ and LKL Problem Solving Group
solutions is April 17, 2010. (Madam Lau Kam Lung Secondary
1 n −1 kn n ⎛ n ⎞ k ( n − 2 j )
Problem 336. (Due to Ozgur Kircak,
= ∑ω ∑
2n k = 0
⎜⎜ ⎟⎟ω
j =0 ⎝ j ⎠
School of MFBM).

Yahya Kemal College, Skopje, Problem 333. Find the largest positive
1 n
⎛ n ⎞ n−1 2 n−2 j k integer n such that there exist n
Macedonia) Find all distinct pairs (x,y)
of integers satisfying the equation
= n
2
∑ ⎜⎜ ⎟⎟∑ (ω
j = 0 ⎝ j ⎠ k =0
)
4-element sets A1, A2, …, An such that
every pair of them has exactly one
x 3 + 2009 y = y 3 + 2009 x. 1 ⎛⎛n⎞ ⎛ n⎞ ⎞ common element and the union of
= ⎜ ⎜ ⎟n + ⎜ ⎟n ⎟
2 n ⎜⎝ ⎜⎝ 0 ⎟⎠ ⎜⎝ n ⎟⎠ ⎟⎠ these n sets has exactly n elements.
Problem 337. In triangle ABC,∠ABC
Solution. LKL Problem Solving
=∠ACB =40°. P and Q are two points n
= . Group (Madam Lau Kam Lung
inside the triangle such that∠PAB =
2n −1 Secondary School of MFBM).
∠ QAC =20° and ∠ PCB = ∠ QCA
=10°. Determine whether B, P, Q are Problem 332. Let ABCD be a cyclic Let the n elements be 1 to n. For i =1 to n,
collinear or not. quadrilateral with circumcenter O. Let BD let si denote the number of sets in which i
bisect OC perpendicularly. On diagonal appeared. Then s1+s2+⋯+sn = 4n. On
Problem 338. Sequences {an} and {bn} average, each i appeared in 4 sets.
AC, choose the point P such that PC = OC.
satisfies a0=1, b0=0 and for n=0,1,2,…,
Let line BP intersect line AD and the
an +1 = 7an + 6bn − 3, circumcircle of ABCD at E and F Assume there is an element, say 1,
bn +1 = 8an + 7bn − 4. respectively. Prove that PF is the appeared in more than 4 sets, say 1 is in
geometric mean of EF and BF in length. A1, A2, …, A5. Then other than 1, the
Prove that an is a perfect square for all remaining 3×5=15 elements must all be
n=0,1,2,… Solution. HUNG Ka Kin Kenneth distinct. Now 1 cannot be in all sets,
(Diocesan Boys’ School) and Abby LEE otherwise there would be 3n+1>n
Problem 339. In triangle ABC,∠ACB (SKH Lam Woo Memorial Secondary elements in the union. So there is a set A6
=90°. For every n points inside the School). not containing 1. Its intersections with
triangle, prove that there exists a
D each of A1, A2, …, A5 must be different,
labeling of these points as P1, P2, …, Pn
yet A6 only has 4 elements, contradiction.
such that F On the other hand, if there is an element
P P + P P + L + P P ≤ AB .
1 2
2
2 3
2
n −1 n
2 2
O appeared in less than 4 sets, then there
E C would be another element appeared in
Problem 340. Let k be a given positive θ more than 4 sets, contradiction. Hence,
integer. Find the least positive integer P θ every i appeared in exactly 4 sets.
N such that there exists a set of 2k+1 A
B
distinct positive integers, the sum of all Suppose 1 appeared in A1, A2, A3, A4.
its elements is greater than N and the Since PC=OC=BC and ΔBCP is similar Then we may assume that A1={1,2,3,4},
sum of any k elements is at most N/2. to ΔAFP, we have PF=AF. A2={1,5,6,7}, A3={1,8,9,10} and A4=
{1,11,12,13}. Hence, n ≥ 13. Assume n
Next, CB = CD = CP implies P is the
***************** ≥ 14. Then 14 would be in a set A5. The
incenter of ΔABD. Then BF bisects
Solutions ∠ABD yielding ∠FAD =∠ADF, call it θ.
other 3 elements of A5 would come from
**************** A1, A2, A3, say. Then A4 and A5 would
(Alternatively, we have∠FAD = ∠PBD
have no common element, contradiction.
Problem 331. For every positive = ½∠PCD. Then
integer n, prove that ∠AFD = 180°−∠ACD Hence, n can only be 13. Indeed, for the
n −1
n = 180°−∠PCD n = 13 case, we can take A1, A2, A3, A4, as
∑ (−1)
k =0
k
cos n ( kπ / n) =
2n −1
. = 180°− 2∠PBD above and A5={2,5,8,11}, A6={2,6,9,12},
= 180°− 2θ. A7={2,7,10,13}, A8={3,5,10,12}, A9={3,
6,8,13}, A10={3,7,9,11}, A11={4,5,9,13},
Solution. Federico BUONERBA Hence, ∠ADF = θ.) Also, we see ∠AFE
(Università di Roma “Tor Vergata”, A12={4, 6, 10,11} and A13={4,7,8,12}.
= ∠BFA and ∠EAF = θ = ∠ADF =∠
Roma, Italy), CHUNG Ping Ngai (La ABF, which imply ΔAFE is similar to
Salle College, Form 6), Ovidiu Other commended solvers: CHUNG
Mathematical Excalibur, Vol. 14, No. 4, Dec. 09-Mar. 10 Page 4

Ping Ngai (La Salle College, Form 6), f is constant, i.e. there exists a real number The inequality in the next example was
HUNG Ka Kin Kenneth (Diocesan c so that for all real y, f(y)=c. Then the very hard. It was proposed by Reid
Boys’ School) and Carlo PAGANO original equation yields c=a(c−x)−c for Barton and appeared among the 2003
(Università di Roma “Tor Vergata”, all real x, which forces a=0 and c=0. IMO shortlisted problems.
Roma, Italy).
Other commended solvers: LKL
Example 6. Let n be a positive integer
Problem 324. (Due to FEI Zhenpeng, Problem Solving Group (Madam Lau
Northeast Yucai School, China) Let x,y Kam Lung Secondary School of MFBM). and let (x1, x2, …, xn), (y1, y2, …, yn) be
∊(0,1) and x be the number whose n-th two sequences of positive real numbers.
digit after the decimal point is the nn-th Let (z1, z2, … , z2n) be a sequence of
digit after the decimal point of y for all positive real numbers such that for all
n =1,2,3,…. Show that if y is rational, 1 ≤ i, j ≤ n, zi+j2 ≥ xiyj. Let M=max{z1,
then x is rational. Olympiad Corner z2, …, z2n}. Prove that
(continued from page 1)
Solution. CHUNG Ping Ngai (La 2
⎛ M + z2 + L+ z2n ⎞ ⎛ x1 + L + xn ⎞⎛ y1 + L+ yn ⎞
Salle College, Form 6), Problem 3. Let a,b,c be complex ⎜ ⎟ ≥⎜ ⎟⎜ ⎟.
⎝ 2n ⎠ ⎝ n ⎠⎝ n ⎠
Since the decimal representation of y is numbers such that for every complex
eventually periodic, let L be the length number z with |z| ≤ 1, we have |az2+bz+c| Solution. (Due to Reid Barton and
of the period and let the decimal ≤ 1. Find the maximum of |bc|. Thomas Mildorf) Let
representation of y start to become X = max{x1, x2, …,xn}
periodic at the m-th digit. Let k be the Problem 4. Let m,n be integers greater
than 1. Let a1 < a2 < ⋯ < am be integers. and
least common multiple of 1,2,…,L. Let
Prove that there exists a subset T of the set Y = min{x1,x2,…, xn}.
n be any integer at least L and nn ≥ m.
of all integers such that the number of
By the pigeonhole principle, there exist elements of T, denoted by |T|, satisfies By replacing xi by xi’=xi/X, yi by yi’=yi/Y
i < j among 0,1,…,L such that ni ≡ nj and zi by zi’= zi/(XY)1/2, we may assume
am − a1 X=Y=1. It suffices to prove
(mod L). Then for all positive integer d, | T |≤ 1 +
we have ni ≡ ni+d(j-i) (mod L). Since k is 2n + 1 M+z2+⋯+z2n ≥ x1+⋯+xn+y1+⋯+yn. (*)
a multiple of j−i and n ≥ L > i, so we and for every i∊{1,2,⋯,m}, there exist Then
have nn ≡ nn+k (mod L). Since k is also a t∊T and s∊[−n,n] such that ai=t+s.
multiple of L, we have (n+k)n+k ≡nn+k ≡ M + z2 +L+ z2n 1 ⎛ x1 +L+ xn y1 +L+ yn ⎞
≥ ⎜ + ⎟,
nn (mod L). Then the n-th and (n+k)-th Problem 5. For n≥3, we place a number 2n 2⎝ n n ⎠
digit of x are the same. So x is rational. of cards at points A1, A2, ⋯, An and O. We
can perform the following operations: which implies the desired inequality by
Other commended solvers: HUNG Ka applying the AM-GM inequality to the
Kin Kenneth (Diocesan Boys’ School) (1) if the number of cards at some point Ai
and Carlo PAGANO (Università di right side.
Roma “Tor Vergata”, Roma, Italy). is not less than 3, then we can remove 3
cards from Ai and transfer 1 card to each To prove (*), we will claim that for any
Problem 335. (Due to Ozgur KIRCAK, of the points Ai−1, Ai+1 and O (here A0=An, r≥0, the number of terms greater than r
Yahya Kemal College, Skopje, An+1=A1); or on the left side is at least the number of
Macedonia) Find all a∊ℝ for which such terms on the right side. Then the
(2) if the number of cards at O is not less k-th largest term on the left side is
the functional equation f: ℝ→ ℝ
than n, then we can remove n cards from greater than the k-th largest term on the
f (x − f ( y)) = a( f (x) − x) − f ( y) O and transfer 1 card to each A1, A2, ⋯, An. right side for each k, proving (*).
for all x, y ∊ℝ has a unique solution. Prove that if the sum of all the cards
For r≥1, there are no terms greater than
placed at these n+1 points is not less than
Solution. LE Trong Cuong (Lam Son 1 on the right side. For r < 1, let A={i:
n2+3n+1, then we can always perform
High School, Vietnam) xi>r}, B={j: yj>r}, A+B={i+j: i∊A,
finitely many operations so that the
j∊B} and C={k: k>1, zk>r}. Let |A|, |B|,
Let g(x) = f(x)−x. Then, in terms of g, number of cards at each of the points is not
|A+B|, |C| denote the number of
the equation becomes less than n+1.
elements in A, B, A+B, C respectively.
g(x−y−g(y))=ag(x)−x.
Problem 6. Let a1, a2, a3, b1, b2, b3 be
Since X=Y=1, so |A|, |B| are at least 1.
Assume f(y)=y+g(y) is not constant. distinct positive integers satisfying
Now xi>r, yj>r imply zi+j>r. So A+B is
Let r, s be distinct elements in the
(n +1)a1n + na2n + (n −1)a3n | (n +1)b1n + nb2n + (n −1)b3n a subset of C. If A is consisted of
range of f(y)=y+g(y). For every real x,
i1<⋯<ia and B is consisted of j1<⋯<jb,
g(x−r) = ag(x)−x = g(x−s). for all positive integer n. Prove that there
then A+B contains
exists a positive integer k such that bi=kai
This implies g(x) is periodic with for i=1,2,3. i1+j1< i1+j2<⋯ < i1+jb < i2+jb <⋯ <ia+jb.
period T=|r−s|>0. Then
Hence, |C| ≥ |A+B| ≥ |A|+|B|−1 ≥ 1. So
ag(x) −x = g(x−y−g(y)) zk>r for some k. Then M>r. So the left
= g(x+T−y−g(y)) side of (*) has |C|+1≥ |A|+|B| terms
= ag(x+T) − (x+T) Max-Min Inequalities greater than r, which finishes the proof
= ag(x)−x−T. (continued from page 2) of the claim.
This implies T=0, contradiction. Thus,

Vous aimerez peut-être aussi